LSAT and Law School Admissions Forum

Get expert LSAT preparation and law school admissions advice from PowerScore Test Preparation.

User avatar
 Dave Killoran
PowerScore Staff
  • PowerScore Staff
  • Posts: 5853
  • Joined: Mar 25, 2011
|
#44114
Complete Question Explanation
(The complete setup for this game can be found here: lsat/viewtopic.php?t=6788)

The correct answer choice is (A)

If M and T are the only employees to attend a session on the first day, we can infer that S must attend sessions on both the second day and the third day. And because T cannot attend a session on the third day, T must attend a session on the second day. Thus, M and T attend a session on the first day, S and T attend a session on the second day, and S attends a session on the third day. This information is sufficient to prove answer choice (A) correct.
 sarae
  • Posts: 80
  • Joined: Aug 10, 2013
|
#10313
I was a bit confused on how answer A is correct. Is it possible that no one attends the investing conference since it doesn't specify that every conference is attended by at least one person?

Thanks!
 Adam Tyson
PowerScore Staff
  • PowerScore Staff
  • Posts: 5153
  • Joined: Apr 14, 2011
|
#10327
sarae,

You are absolutely right - there is no rule in that game requiring anyone to attend a session on investing. It's possible that M, S and T each attend one session on hiring and one on regulations, and the investing sessions remain blank (or, if you used some new variable like X as a placeholder for "empty" slots, there could be an X in each of the three slots for investing).

The key here is the local rule that sets up the question. According to the question stem, M and T are the only employees that attend any session on Day 1. That means two things - 1) they both have to attend a session on Day 1, and 2) S cannot attend a session on Day 1. Where does that leave our friend S? S will have to attend one session on hiring and one on regulations, and those will both be on days 2 and 3.

Remember that this is a "Could Be True - Except" question - the correct answer will be something that Cannot Be True. Answer choice A is asking us if we can have M and S attend two sessions together. The global rules are still in effect here, so we know that each person attends exactly two sessions, and M attends one session on Day 1, where S cannot be. Could M and S attend two sessions together? Not a chance - M only gets to go to one more session, other than whatever she attends on Day 1. Maybe it's with S, maybe it isn't, but either way those two cannot do two sessions together.

Hope that helped! I can see where the explanation in the book might leave you wondering, but try working through it again and it should come clear.

Good luck and enjoy your class!
 sarae
  • Posts: 80
  • Joined: Aug 10, 2013
|
#10349
That helped a lot! Thank you so much!!

Get the most out of your LSAT Prep Plus subscription.

Analyze and track your performance with our Testing and Analytics Package.